LSAT and Law School Admissions Forum

Get expert LSAT preparation and law school admissions advice from PowerScore Test Preparation.

 Administrator
PowerScore Staff
  • PowerScore Staff
  • Posts: 8917
  • Joined: Feb 02, 2011
|
#40017
Complete Question Explanation
(The complete setup for this game can be found here: lsat/viewtopic.php?t=12912)

The correct answer choice is (E)

To attack this question, use proper List question technique: take one rule and apply it to all answer choices, then take another rule, and apply it to the remaining answer choices, and so on, until only one answer choice remains.

Answer choice (A): This answer choice violates the third rule and is therefore incorrect.

Answer choice (B): This answer choice violates the first rule and is therefore incorrect.

Answer choice (C): This answer choice violates the second rule and is therefore incorrect.

Answer choice (D): This answer choice violates the fourth rule and is therefore incorrect.

Answer choice (E): This is the correct answer choice.

As usual, each of the four incorrect answer choices violates a different rule.
 k100
  • Posts: 10
  • Joined: Nov 21, 2019
|
#72187
Hello,

If the contrapositive of Gs :arrow: Lt is (not) Lt :arrow: (not) Gs, how can G be in t if L is in t? Wouldn't this represent "Lt :arrow: (not) Gs"?

Thanks!
User avatar
 Dave Killoran
PowerScore Staff
  • PowerScore Staff
  • Posts: 5852
  • Joined: Mar 25, 2011
|
#72190
Hi K100,

Thanks for the question! Isolating this rule, is it possible for G and L to both be at T? Yes. Here's why:

  • If you put G at T, how does that affect this rule? It doesn't, because there is no sufficient condition for G being at T (or alternately, there is no necessary condition that requires G being at S.

    You asked about this rule: Lt :arrow: (not) Gs. But that's not actually a rule :-D It's neither the way the rule was originally stated nor its contrapositive, so it doesn't apply!
Please let me know if it helps!
 k100
  • Posts: 10
  • Joined: Nov 21, 2019
|
#72200
Hi Dave,

Thanks for your response!

So my question was about how G and L can be at T, because the contrapositive of rule #3 is "if L is not in T, then G is not in S"; however in the correct answer, L is in T and G is not in S. I misinterpreted this to mean that if G is not in S, it must be in T, but does it actually mean that G can be in T, or neither S nor T (since not all photographers have to be assigned)?
 Zach Foreman
PowerScore Staff
  • PowerScore Staff
  • Posts: 91
  • Joined: Apr 11, 2019
|
#72217
k100,
The original rule is Gs -->Lt
The contrapositive is Lt --> Gs

So, the diagramming is fine, it is the interpretation that is hard. The key is not to infer too much. The sufficient conditions are the only things that trigger or force something to happen. So, if G is in S or if L is NOT in T then something has to happen. So, let's take your case of L being in T. The original sufficient condition is not triggered because it is about G. The cp sufficient condition is not triggered because it is about L NOT being in T. So, no inference, we cannot know anything else if all we know is that L is in T. Saying that if L is in T therefore G must be in S would be a Mistaken Reversal of the original, not a contrapositive.
Now, the only other thing to say is that if the game were set up so that every photographer MUST be at one or the other school and no one could be at both, then the contrapositive could be written Ls --> Gt. But that is not true in this game.
 k100
  • Posts: 10
  • Joined: Nov 21, 2019
|
#72238
Ok, that really clears things up. Thanks a lot!

Get the most out of your LSAT Prep Plus subscription.

Analyze and track your performance with our Testing and Analytics Package.